Answer:
Ummm WHAT! IS THIS FREE POINTS?!
Step-by-step explanation:
como se representa un numero par
hay numero pares e impares que son enteros y los pares son tosodo los que se pueden dividir entre dos siempre y cuando de numeros enteros
In Problem, p is in dollars and q is the number of units.
(a) Find the elasticity of the demand function
p2 + 2p + q = 49 at p = 6.
(b) How will a price increase affect total revenue?
Answer:
-14
Explanation:
Elasticity of demand is the degree of change in demand after a change I'm price, basically demand's sensitivity to price change.
Formula for calculating price elasticity is: change in price/change in quantity =dq/dp
Since we are given p²+2p+q=49 and not initial and current amount of price and quantity, we differentiate to find demand elasticity, thus:
2p+2+dq/dp=0
dq/dp=-2p-2
Given p =6, we substitute:
dq/dp=-2×6-2
dq/dp=-12-2
dq/dp=-14
With a demand elasticity of -14 there is an inverse relationship between price and demand. While price increases, demand falls.
Express it in slope-Intercept form
Answer:
Y=1/4x-4
Explanation: The y intercept is -4 that is your B. Using the rise over sun method the line rises 1 and goes to the right 4 making the slope 1/4 or .25
in the diagram, four circles of radius 1 with centers $p$, $q$, $r$, and $s$ are tangent to one another and to the sides of $\triangle abc$, as shown.
The answer is 8 + 8√2 = 12 + 4√2, which is closest to answer choice (A), so the answer is (A).
Since the circles have radius 1, the sides of the triangle that are tangent to the circles are equal to 2. The sum of the sides of the triangle is equal to the perimeter of the triangle, so if we add up the sides that are tangent to the circles, we can find the perimeter.
There are four sides of the triangle that are tangent to the circles, so the sum of these sides is
4 * 2 = 8
To find the perimeter, we simply add 8 to the remaining sides of the triangle.
Since the triangle is isosceles, it has two equal sides that are equal to 4√2, and one side equal to the diameter of the circles, which is equal to 4. The perimeter is then equal to :
8 + 4 + 4√2 * 2 = 8 + 4 + 8√2.
The answer is 8 + 8√2 = 12 + 4√2, which is closest to answer choice (A), so the answer is (A).
Learn more about tangent :
https://brainly.com/question/19424752
#SPJ4
The complete question is :
In the diagram, four circles of radius 1 with centers P, Q, R and S are tangent to one another and sides of △ABC . Find the perimeter of △ABC
(A) 12+4√2
(B) 12+4√3
(C) 12+6√2
(D) 12+6√3
(E) 24
Which one of the following numbers is an irrational number?
Question 20 options:
A)
134∕675
B)
3.
C)
7.676767…
D)
8.12131415…
Answer:
B
Step-by-step explanation:
rational number means comparison between two numbers
eg: 1:3 or 1/3 or 0.333
irritional number does not
The one of the numbers is an irrational number that is A) 134 / 675
What are rational numbers, and irrational numbers?Rational numbers are numbers which can be written in the form of \(\dfrac{a}{b}\)where a and b are integers. Example: 1/2, 3.5 (which is writable as 7/5) etc.
Irrational numbers are those real numbers which are not rational numbers.
Noted that all natural numbers are integers, all integers are rational numbers. That means, natural numbers are not irrational.
For example
: 1:3 or 1/3
or 0.333
But irrational number does not.
The rational number means the comparison between two numbers
A) 134 / 675 = 0.19
B) 3.
C) 7.676767…
D) 8.12131415…
So, A) 134 / 675
This is an irrational number
Learn more about numbers here;
https://brainly.com/question/3151755
#SPJ2
how to find area of irregular shapes?
Finding the area of irregular shapes can be challenging, as they do not have a regular geometric formula to calculate their area. However, there are a few methods you can use depending on the shape and available information. Here are three common approaches:
Break it into simpler shapes: If the irregular shape can be divided into simpler geometric shapes (such as rectangles, triangles, circles, or trapezoids), you can calculate the area of each individual shape and then sum them up to find the total area. This method is called the “composite shape” or “dissection” method.Approximation using grids: If you have a grid or graph paper, you can overlay it on the irregular shape and count the number of squares that are fully or partially covered by the shape. Multiply the number of squares by the area of each square to estimate the area of the irregular shape. This method is known as the “grid method” or “counting squares.”Using calculus: If you have an equation or a graph of an irregular shape, you can use calculus techniques to find the area. By integrating the function that represents the shape's boundary or using numerical integration methods, you can determine the area under the curve. This method is more advanced and requires knowledge of calculus.It's important to note that these methods provide approximations and may not yield exact results, especially for highly complex irregular shapes. Additionally, there may be specialized techniques for specific types of irregular shapes. In some cases, using computer software or online tools designed for area calculations can also be helpful.
Remember to carefully assess the shape and available information to choose the most suitable method for finding the area of an irregular shape in your specific situation.
HELPP PLEASE!!!! SHOW ALL WORK PLEASE
Answer:
#1 is 2x-12°=-12
#2 is x+34°=34
Step-by-step explanation:
when it tells you to add the second number will always be that number but a positive.
when it tells you to subtract the second number will always be that number but a negative.
plus=positive
minus=negative
remember that.
hope that helped you out!
Find theValue of x
40°
70°
(5x+10)°
Value of an exterior angle of a triangle is equal to the sum of values of two opposite interior angles of a triangle.
therefore,\(\qquad\displaystyle \tt \dashrightarrow \: 5x + 10 = 40 + 70\)
\(\qquad\displaystyle \tt \dashrightarrow \: 5x = 110 - 10\)
\(\qquad\displaystyle \tt \dashrightarrow \: 5x = 100\)
\(\qquad\displaystyle \tt \dashrightarrow \: x = 100 \div 5\)
\(\qquad\displaystyle \tt \dashrightarrow \: x = 20\)
Value of x = 20°
Hey! there . Thanks for your question :)
Answer:
20° is the correct answer.Step-by-step explanation:
In this question we are given with two interior angles of the triangle that are 40° and 70° , also we are given an exterior angle that is (5x + 10)°. And we are asked to find the value of angle x.
Solution :-
For finding the value of angle x , we have to use exterior angle property of triangle which states that sum of opposite interior angles of triangle is equal to the given exterior angle. So :
Step 1: Making equation :
\( \longmapsto \: \sf{40 {}^{°} + 70 {}^{°} = (5x + 10) {}^{°} }\)
Solving :
\( \longmapsto \: \sf{110 {}{°} = (5x) {}^{°} +10 {}^{°} }\)
Step 2: Subtracting 10 on both sides :
\( \longmapsto \sf{ 110 {}^{°} - 10 {}^{°} = 5x + \cancel{10 {}^{°}} - \cancel{10 {}^{°} } }\)
We get ,
\( \longmapsto \sf{(5x ){}^{°} = 100 {}^{°} }\)
Step 3: Dividing both sides by 5 :
\( \longmapsto \dfrac{ \cancel{5}x {}^{°} }{ \cancel{5}} = \dfrac{ \: \: \: \: \cancel{ 100} {°}^{} }{ \cancel{5} }\)
On cancelling , we get :
\( \longmapsto \underline{\boxed{\red{\sf{ \bold{ x = 20 {}^{°} }}}}} \: \: \bigstar\)
Therefore , value of x is '20°'Verification :-
For verifying sum of both the interior angles is equal to given exterior angles. As we get the value of x as 20 we need to substitute it's value in place x and then L.H.S must be equal to R.H.S :
40° + 70° = 5(20°) + 10°110° = 100° + 10°110° = 110°L.H.S = R.H.STherefore , our answer is correct .
Hope , it'll help you! :)#\( \underline{ \sf{ \bold{ Keep \: Learning }}}\)If someone build a house for 50 years how many months did it take to complete
Answer: 600
Step-by-step explanation: 12 months in one year. 12x50=600 months
Answer:
600 months
Step-by-step explanation:
In 1 year there are 12 months, so we take 50 (the years) and multiply that by 12
50x12=600 and we fact check by 600÷12=50. It takes 600 months to build the house over 50 years.
Convert the following polar-form vectors of the form ⟨r,θ⟩ into component-form vectors of the form ⟨x,y⟩. Your answers should be in the form "< #, # >". ⟨13,50∘⟩= ⟨1.77,250∘⟩= ⟨5.8,150∘⟩=
9514 1404 393
Answer:
a) <8.356, 9.959>
b) <-0.605, -1.663>
c) <-5.023, 2.9>
Step-by-step explanation:
Many calculators can perform polar ⇔ rectangular conversion. Attached is the result from one of them. Of course, you can also program a spreadsheet to do it. (The ATAN2( ) function is useful for finding the correct angle.)
If you want to do these calculations by hand, the conversion is ...
<r, θ> ⇒ <r·cos(θ), r·sin(θ)>
In the attached, the rectangular coordinates are shown as complex numbers. The imaginary component is the y-component of the vector.
Apply
14. The table shows the record high temperature, in degrees
Fahrenheit (°F) or degrees Celsius (°C), of certain states. The
formula F = 1.8C + 32 can be used to convert between degrees
Celsius and degrees Fahrenheit. What is the difference, in
degrees Celsius, between Nevada's record high temperature
and Alaska's record high temperature? Round to the nearest
tenth.
State
Alaska
Florida
Nevada
Texas
Record High
Temperature
38°C
109°F
125°F
49°C
Point A is located at (1, 2). Point B is located at (4, 6). Use this information to determine the length of the line, rounded to the nearest whole number.
If Point A is located at (1, 2) and Point B is located at (4, 6), the length of the line between points A and B is 5 units.
To determine the length of the line between points A and B, we can use the distance formula, which is a formula used to calculate the distance between two points in a coordinate plane. The distance formula is:
d = √((x₂ - x₁)² + (y₂ - y₁)²)
where (x₁, y₁) and (x₂, y₂) are the coordinates of the two points and d is the distance between them.
Using the coordinates of points A and B, we can substitute their values into the distance formula to find the length of the line between them:
d = √((4 - 1)² + (6 - 2)²)
d = √(3² + 4²)
d = √(9 + 16)
d = √25
d = 5
Rounded to the nearest whole number, the length of the line is also 5 units.
In conclusion, we can use the distance formula to find the length of the line between two points in a coordinate plane. The distance formula uses the coordinates of the two points to calculate the distance between them. The resulting distance can be rounded to the nearest whole number, if needed.
To learn more about distance click on,
https://brainly.com/question/28658621
#SPJ1
If my dog is yellow and red on the tail blue on it’s left foot how much does he weigh and is he orange?
seventy five pounds
Step-by-step explanation:
because the colors make him heavy duh
2(10) + 2(x – 4) please simplify the expression
options
1. 2x + 16
2. x + 12
3. 2x + 12
4. x + 16
Answer:
3
Step-by-step explanation:
Juan and Adam both need pens and
batteries. The equation that represents
Juan's purchases is y - 27 - 3x. The
equation that represents Adam's purchases
is y = 17 – x. If y represents the price of the
batteries, and x represents the price of the
pens, what are the prices of the pens and
batteries?
a) pens: $10 batteries: $7
b) pens: $5 batteries: $12
o pens: $7 batteries: $20
d) pens: $12 batteries: $20
e) pens: $5 batteries: $10
go to station 4
go to station 3
go to station 7
go to station 9
go to station 2
Answer:
a
Step-by-step explanation:
Cameron and Cadena go out to eat
for lunch.
Part A
If their food and beverages cost
$22.50 and there is a 9% meals tax,
how much is the bill? Write your
answer to the nearest cent.
Part B
After adding tax and a tip, the
total lunch cost was $28.95. What
percentage of the cost of their food and
beverages did they give as tip? Write
your answer to the nearest percentage
a. The bill when Cameron and Cadena go out to eat for lunch is $24.525
b. The percentage of the cost of their food and beverages that they give as tip will be 18%.
How to calculate the value?From the information, Cameron and Cadena go out to eat for lunch. If their food and beverages cost $22.50 and there is a 9% meals tax, the bill will be:
= $22.50 + (9% × $22.50)
= $22.50 + $2.025
= $24.525
The percentage of the cost of their food and beverages that they give as tip will be:
= (28.95 - 24.525) / 24.525 × 100
= 18%
Learn more about percentages on:
brainly.com/question/24877689
#SPJ1
4x - 3 = 17
-2x - 7y = 11
What is F(-x)?
F(x)=x^3+1
Answer:
F(-x) = 0
Step-by-step explanation:
What is F(-x)?
F(-x) = F(-1)
F(x) = x³ + 1 F(-1)
F(-1) = -1³ + 1
F(-1) = -1 + 1
F(-1) = 0
So, F(-x) = 0
Answer:
f(-1) = 0
Step-by-step explanation:
Evaluating a function for f(-x) is the same as evaluating it for f(-1).
So we plug in -1 instead.
f(-1) = x³ + 1
= (-1)³ + 1 (here we cube -1)
= -1 + 1 (here we subtract)
= 0 (and, this is the answer)
\(\therefore\) The answer is f(-1) = 0
(n-1 +7) simplify plssss helppp
Answer: n+6
-1 + 7 = 6
So therefore, the answer is n+6
Hope this helped
Answer:
\(\boxed{\sf{n+6}}\)Step-by-step explanation:
To solve this problem, first, you have to do is isolate it on one side of the equation.
(n-1+7)First, remove parentheses.
→ n-1+7
Add the numbers from left to right.
→ -1+7=6
→ n+6
Therefore, the final answer is n+6.I hope this helps you! Let me know if my answer is wrong or not.
Myron records the number of chirps per minute (x) that crickets make at
different temperatures (v) in degrees Fahrenheit.
He determines that the association is linear and that the line of best fit is
y=+50.
What is the interpretation of the slope and y-intercept of this equation?
A. The slope predicts a temperature increase of about 50° for every
increase of 1 chirp per minute. The y-intercept shows that when
the crickets are not chirping (x = 0), the temperature is 0°.
B. The slope predicts a temperature increase of about 6* for every
increase of 50 chirps per minute. The y-intercept shows that when
the crickets are not chirping (x= 0), the temperature is
OC. The slope predicts a temperature increase of about for every
increase of 1 chirp per minute. The y-intercept shows that when
the crickets are not chirping (x = 0), the temperature is 50°.
D. The chirping increases as the temperature goes down.
The correct interpretation of the slope and y-intercept of the equation
y = 50 is:
A. The slope predicts a temperature increase of about 50° for every increase of 1 chirp per minute. The y-intercept shows that when the crickets are not chirping (x = 0), the temperature is 0°.
In the given equation y = 50, the slope of 50 indicates that for every increase of 1 chirp per minute (x), the temperature (y) increases by approximately 50° Fahrenheit.
The positive slope suggests a positive correlation between the number of chirps per minute and temperature.
The y-intercept of 0 in this equation represents the temperature when the crickets are not chirping (x = 0). It implies that when there are no chirps per minute, the temperature is at 0° Fahrenheit.
Learn more about Slope here:
https://brainly.com/question/3605446
#SPJ1
please answer i’ll give brainliest !!!
there are 25 student in a class. five of then scored A and 10 of them score B while the other scored C for Biostatistics. if a student is selected at random, calculate the probability that the selected student scored A or B in biostastics.
There is a 60% probability that a randomly selected student from the class scored either an A or B in Biostatistics.
To calculate the probability that a randomly selected student scored either an A or B in Biostatistics, we need to consider the number of students who scored A and B and divide it by the total number of students in the class.
Given that there are 25 students in the class, 5 of them scored an A and 10 scored a B. To calculate the probability, we add the number of students who scored A (5) to the number of students who scored B (10):
Number of students who scored A or B = Number of students who scored A + Number of students who scored B = 5 + 10 = 15.
Therefore, the probability that a randomly selected student scored A or B
in Biostatistics is:
Probability = Number of students who scored A or B / Total number of students = 15 / 25 = 0.6 or 60%.
for more such questions on probability
https://brainly.com/question/25839839
#SPJ8
Use the equation and type the ordered pairs. y = log_2 x
Given:
\(y=\log _2x\)When x=1,
\(y=\log _21\)\(y=0\)When x= 2,
\(y=\log _22\)\(y=1\)When x= 4,
\(y=\log _24\)\(y=2\)When x=8,
\(y=\log _28\)\(y=3\)When x=16,
\(y=\log _216\)\(y=4\)Ordered pairs are
\((1,0),(2,1),(4,2),(8,3)\text{ and (16,4)}\)I will give brainlist
One third times the difference of thirty and a variable is three fourths.
one third times the quantity 30 minus y equals three fourths
one third times the quantity y minus 30 equals three fourths
one third times 30 minus y equals three fourths
one third times y minus 30 equals three fourths
It should be noted that one third times the difference of thirty and a variable is three fourths will be D. one third times y minus 30 equals three fourths
How to illustrate the information?It should be noted that an expression simply shows that relationship between the numbers and the variables that are explained.
In this case, it should be noted that one third times the difference of thirty and a variable is three fourths will be one third times y minus 30 equals three fourths
Therefore, the appropriate explanation that first the information illustrated is D.
Learn more about expressions on:
brainly.com/question/25968875
#SPJ1
Answer:
The guy above me is correct its D :)
Step-by-step explanation:
please don't delete this again :)
have a goody day
ma'am or sir
help me! I have a retake on this and my last answer I put was A and D and I got it wrong!
9514 1404 393
Answer:
A, E
Step-by-step explanation:
You are correct that the appropriate relationship is ...
g = 0.4t . . . . choice A
You are also correct that an equivalent relationship is ...
t = 2.5g . . . . choice D
However, when we say gallons are proportional to seconds, we usually expect that to be written ...
g = kt . . . . for some value of k
__
We note that another equivalent is ...
g = 2/5t . . . . choice E
The appropriate choices are probably A and E. Choice D might be argued to be correct, but you could very well lose that argument to someone who prefers a "g = " form of the relationship.
Audrey went shopping and bought two pairs of
jeans for $16.95 each, a shirt for $16.75, and a
belt for $9.50. If she paid for the purchases with
a $100 bill, how much change should she get?
The change that she should get in return will be $39.85.
What is Algebra?Algebra is the study of abstract symbols, while logic is the manipulation of all those ideas.
The acronym PEMDAS stands for Parenthesis, Exponent, Multiplication, Division, Addition, and Subtraction. This approach is used to answer the problem correctly and completely.
Audrey went shopping and bought two pairs of jeans for $16.95 each, a shirt for $16.75, and a belt for $9.50. If she paid for the purchases with a $100 bill.
Then the change that she should get will be given as,
Change = $100 - 2 × $16.95 - $16.75 - $9.50
Change = $100 - $33.90 - $16.75 - $9.50
Change = $100 - $60.15
Change = $39.85
The change that she should get in return will be $39.85.
More about the Algebra link is given below.
https://brainly.com/question/953809
#SPJ2
Use the graph of g to find g(x) = 3.
pls help solve quick!
By using the graph of g, the solution to g(3) is equal to 8.
What is a function?In Mathematics and Geometry, a function can be defined as a mathematical expression which is typically used for defining and representing the relationship that exists between two or more variables such as an ordered pair in tables or relations.
What is a domain?In Mathematics and Geometry, a domain is sometimes referred to as input value and it can be defined as the set of all real numbers for which a particular function is defined.
When the domain (input value) of the given function g(x) shown in the graph is 3, the output value (range) is given by;
g(3) = 8.
Read more on function here: brainly.com/question/9795474
#SPJ1
Missing information:
The question is incomplete and the complete question is shown in the attached picture.
In triangle ABC, the measure of angle B is 33 more than three times the measure of angle A. The measure of angle C is 57 more than the measure of angle A. Find the measure of each angle.
Answer:
a= 18
b=87
c=75
Step-by-step explanation:
1. We will use Gaussian elimination to solve this problem.
b= 33+3a
c= 57+a
a+b+c= 180
2. Swap Row1 and Row3 to make row reduction easier.
a+b+c=180
-a+c=57
-3a+b=33
3. Apply to Row2 : Row2 + Row1.
a+b+c=180
b+2c=237
-3a+b=33
4. Apply to Row3 : Row3 + 3 Row1.
a+b+c=180
b+2c=237
4b+3c=573
5. Apply to Row3 : Row3 - 4 Row2.
a+b+c=180
b+2c=237
-5c=-375
6. Simplify rows.
a+b+c=180
b+2c=237
c=75
7. Apply to Row2 : Row2 - 2 2 Row3.
a+b+c=180
b=87
c=75
8. Apply to Row1 : Row1 - Row3.
a+b=105
b=87
c=75
9. Apply to Row1 : Row1 - Row2.
a=18
b= 87
c= 75
10. Simplify rows.
a=18
b= 87
c=75
29.For n ≥ 3, a pattern can be made by overlapping n circles, each of circumference 1 unit, so that each circle passes through a central point and the resulting pattern has order-n rotational symmetry.
For instance, the diagram shows the pattern where n = 7.
If the total length of visible ares is 60 units, what is n?
The value of n can be determined by finding the number of visible arcs in the pattern, which is 30 in this case.
To determine the value of n, we need to find the relationship between the total length of visible areas and the number of circles (n).
In the given pattern, each circle contributes to the visible area twice: once as its circumference and once as the overlapping part with the adjacent circles. Since the circumference of each circle is 1 unit, the visible area contributed by each circle is 2 units.
Therefore, the total length of visible areas can be expressed as 2n. Given that the total length is 60 units, we can set up the equation:
2n = 60
Solving this equation, we find:
n = 60/2 = 30
Thus, the value of n is 30.
For more questions on number of visible arcs:
https://brainly.com/question/31336038
#SPJ8
According to a survey about 82% of a highly satisfied with their jobs at 45 engineers were surveyed how many reported that they were highly satisfied?